bold face fever

This topic has expert replies
Legendary Member
Posts: 1578
Joined: Sun Dec 28, 2008 1:49 am
Thanked: 82 times
Followed by:9 members
GMAT Score:720

bold face fever

by maihuna » Sun Jan 18, 2009 9:45 am
Letter to the editor: Proposition Q, a controversial measure on this year’s ballot, would prohibit the ownership of handguns within the city’s limits. Under the plan, gun owners would have a 90-day grace period to turn in their weapons to authorities. Proponents of the proposition argue that fewer handguns on the streets would lead to less violent crime, making the city safer for all of its citizens. Unfortunately, the ban would actually have the opposite effect. Since only law abiding citizens would honor the ban, armed criminals would not only keep their weapons but would also have the confidence to act with impunity on a population that could no longer defend itself.

In the letter to the editor, the two portions in boldface play which of the following roles?
The first is an observation that the author uses to support a particular position; the second is that position.
The first is a pattern of cause and effect that the author believes to be true; the second offers evidence to contradict this pattern.
The first is a position that the author argues will not hold in this case; the second is the author’s position.
The first is a prediction that the author believes to be untrue. The second is a statement of fact that undermines the author’s position.
The first is a direct relationship that the author believes will not hold in this case; the second offers evidence in support of the author’s position.
Last edited by maihuna on Mon Jan 19, 2009 8:15 am, edited 1 time in total.

Senior | Next Rank: 100 Posts
Posts: 36
Joined: Sat Jul 26, 2008 10:08 am
Location: LA,US

by bec.amit » Sun Jan 18, 2009 10:18 am
IMO E

Master | Next Rank: 500 Posts
Posts: 344
Joined: Sun Sep 28, 2008 11:00 am
Location: USA
Thanked: 6 times
Followed by:1 members

by Bidisha800 » Sun Jan 18, 2009 12:53 pm
bec.amit wrote:IMO E
how did you answer when the question is not even bold-faced ?
Drill baby drill !

GMATPowerPrep Test1= 740
GMATPowerPrep Test2= 760
Kaplan Diagnostic Test= 700
Kaplan Test1=600
Kalplan Test2=670
Kalplan Test3=570

Senior | Next Rank: 100 Posts
Posts: 36
Joined: Sat Jul 26, 2008 10:08 am
Location: LA,US

by bec.amit » Sun Jan 18, 2009 7:43 pm
The answer speaks of two parts.(Actually just took the benefit of doubt)

Master | Next Rank: 500 Posts
Posts: 163
Joined: Fri Sep 26, 2008 10:54 pm
Thanked: 7 times

by jeevan.Gk » Tue Jan 20, 2009 3:31 am
IMO C ! ..

What is the OA

Junior | Next Rank: 30 Posts
Posts: 25
Joined: Thu Jan 08, 2009 8:19 am
Thanked: 1 times

by gg1 » Tue Jan 20, 2009 7:12 am
E

Conclusion : ban will have opposite effect.

1st will not hold in this case.
2nd statement is an evidence for the conclusion.

Master | Next Rank: 500 Posts
Posts: 163
Joined: Fri Sep 26, 2008 10:54 pm
Thanked: 7 times

by jeevan.Gk » Fri Jan 30, 2009 11:19 pm
HI.. Pls post the OA.. Is the answer not C!

Legendary Member
Posts: 727
Joined: Sun Jun 08, 2008 9:32 pm
Thanked: 8 times
Followed by:1 members

by umaa » Sat Jan 31, 2009 9:09 am
IMO E. First says, "argue that", which means the author is not supporting it. Second is author's statement. So, A, B and D are out.

C - Second is not author's position. Author is providing an evidence which is supporting his position.

So, E.

OA pls.